JBMO Τέστ Εξάσκησης #2

Συντονιστές: cretanman, ΔΗΜΗΤΡΗΣ ΙΩΑΝΝΟΥ, socrates

achilleas
Γενικός Συντονιστής
Δημοσιεύσεις: 3014
Εγγραφή: Τρί Σεπ 15, 2009 3:32 pm

JBMO Τέστ Εξάσκησης #2

#1

Μη αναγνωσμένη δημοσίευση από achilleas » Σάβ Ιουν 15, 2019 2:24 pm

Καλησπέρα σας!

Σε συνέχεια του προηγούμενου θέματος, ακολουθούν τα προβλήματα του 2ου τεστ.

Θα χαρούμε να δούμε κι άλλες διαφορετικές λύσεις στα παρακάτω θέματα.

**********************************************

JBMO Practice TEST 2
ΔΙΑΡΚΕΙΑ: 4,5 ΩΡΕΣ

ΘΕΜΑ 1. Έστω O το περίκεντρο του τριγώνου ABC. Θεωρούμε τον κύκλο k ο οποίος βρίσκεται στο ημιεπίπεδο της ευθείας BC που δεν περιέχει το A και εφάπτεται στην πλευρά BC και τις ευθείες CA και AB στα σημεία D, E και F, αντίστοιχα. Αν η ακτίνα του k ισούται με την ακτίνα του περιγεγραμμένου κύκλου του τριγώνου ABC, να αποδείξετε ότι οι ευθείες OD και EF είναι μεταξύ τους κάθετες.

ΘΕΜΑ 2. Να δειχθεί ότι η εξίσωση \displaystyle  
	x^2+y^2+z^2=x+y+z+1 
	δεν έχει ρητές λύσεις.

ΘΕΜΑ 3. Να δειχθεί ότι αν οι a,b,c είναι θετικοί αριθμοί τέτοιοι ώστε a^2+b^2+c^2=3, τότε \displaystyle  
	\dfrac{1}{a}+\dfrac{3}{b}+\dfrac{5}{c}\geq 4a^2+3b^2+2c^2.
Πότε ισχύει η ισότητα?

ΘΕΜΑ 4. Σε n διαφανή κουτιά υπάρχουν κόκκινες και μπλε μπάλες. Θέλουμε να επιλέξουμε 50 κουτιά τέτοια ώστε να περιέχουν, μαζί, τουλάχιστον τις μισές κόκκινες και τουλάχιστον τις μισές μπλε μπάλες. Είναι μια τέτοια επιλογή δυνατή ανεξάρτητα από τον αριθμό των μπαλών και από την κατανομή τους στα κουτιά, αν

(α) n=100?
(β) n=99?

**********************************************

Φιλικά,

Αχιλλέας



Λέξεις Κλειδιά:
Xriiiiistos
Δημοσιεύσεις: 219
Εγγραφή: Τρί Μάιος 15, 2018 4:36 pm

Re: JBMO Τέστ Εξάσκησης #2

#2

Μη αναγνωσμένη δημοσίευση από Xriiiiistos » Σάβ Ιουν 15, 2019 5:35 pm

achilleas έγραψε:
Σάβ Ιουν 15, 2019 2:24 pm

ΘΕΜΑ 3. Να δειχθεί ότι αν οι a,b,c είναι θετικοί αριθμοί τέτοιοι ώστε a^2+b^2+c^2=3, τότε \displaystyle  
	\dfrac{1}{a}+\dfrac{3}{b}+\dfrac{5}{c}\geq 4a^2+3b^2+2c^2.
Πότε ισχύει η ισότητα?
Iσοδύναμα αρκεί να δείξω b^{2}+2c^{2}+\frac{1}{a}+\frac{3}{b}+\frac{5}{c}\geq 12 και το αριστερό μέλος γράφεται b^{2}+c^{2}+c^{2}+\frac{1}{a}+\frac{1}{b}+\frac{1}{b}+\frac{1}{b}+\frac{1}{c}+\frac{1}{c}+\frac{1}{c}+\frac{1}{c}+\frac{1}{c}\geq 12\sqrt[12]{\frac{1}{abc}}\geq 12 αφού a^{2}+b^{2}+c^{2}=3\geq 3\sqrt[3]{(abc)^{2}} ισότητα όταν a=b=c=1 ...


Άβαταρ μέλους
george visvikis
Επιμελητής
Δημοσιεύσεις: 13298
Εγγραφή: Παρ Νοέμ 01, 2013 9:35 am

Re: JBMO Τέστ Εξάσκησης #2

#3

Μη αναγνωσμένη δημοσίευση από george visvikis » Σάβ Ιουν 15, 2019 5:39 pm

achilleas έγραψε:
Σάβ Ιουν 15, 2019 2:24 pm

JBMO Practice TEST 2
ΔΙΑΡΚΕΙΑ: 4,5 ΩΡΕΣ

ΘΕΜΑ 1. Έστω O το περίκεντρο του τριγώνου ABC. Θεωρούμε τον κύκλο k ο οποίος βρίσκεται στο ημιεπίπεδο της ευθείας BC που δεν περιέχει το A και εφάπτεται στην πλευρά BC και τις ευθείες CA και AB στα σημεία D, E και F, αντίστοιχα. Αν η ακτίνα του k ισούται με την ακτίνα του περιγεγραμμένου κύκλου του τριγώνου ABC, να αποδείξετε ότι οι ευθείες OD και EF είναι μεταξύ τους κάθετες.
JBMO Test 2.png
JBMO Test 2.png (19.39 KiB) Προβλήθηκε 1648 φορές
Προφανώς ο (k) είναι ο A-παρεγγεγραμμένος κύκλος του τριγώνου ABC. Αν η AI_a τέμνει τον κύκλο (O) στο M,

τότε OM\bot BC και AI_a\bot EF κι επειδή R=r_a, το OMI_aD θα είναι παραλληλόγραμμο. Άρα \boxed{OD\bot EF}


Άβαταρ μέλους
Ορέστης Λιγνός
Δημοσιεύσεις: 1835
Εγγραφή: Κυρ Μάιος 08, 2016 7:19 pm
Τοποθεσία: Χαλάνδρι Αττικής
Επικοινωνία:

Re: JBMO Τέστ Εξάσκησης #2

#4

Μη αναγνωσμένη δημοσίευση από Ορέστης Λιγνός » Σάβ Ιουν 15, 2019 11:06 pm

achilleas έγραψε:
Σάβ Ιουν 15, 2019 2:24 pm
Καλησπέρα σας!

Σε συνέχεια του προηγούμενου θέματος, ακολουθούν τα προβλήματα του 2ου τεστ.

Θα χαρούμε να δούμε κι άλλες διαφορετικές λύσεις στα παρακάτω θέματα.

**********************************************



ΘΕΜΑ 2. Να δειχθεί ότι η εξίσωση \displaystyle  
	x^2+y^2+z^2=x+y+z+1 
	δεν έχει ρητές λύσεις.
Γνωστό θέμα.

Ισοδύναμα, θέλω η (2x-1)^2+(2y-1)^2+(2z-1)^2=7 να μην έχει ρητές λύσεις, ή θέτοντας 2x-1=k/q, 2y-1=\ell/q, 2z-1=m/q με (k,\ell,m,q)=1 αρκεί να δείξω ότι η k^2+\ell^2+m^2=7q^2 δεν έχει ακέραιες λύσεις.

Αν q άρτιος, τότε πρέπει αναγκαστικά δύο εκ των k,\ell,m να είναι περιττοί και ένας άρτιος. Άρα, \pmod 8, LHS \equiv 2 ή 6 \pmod 8 και RHS \equiv 7 ή 28 \pmod 8, άτοπο.

Αν q περιττός, τότε αν οι k,\ell,m είναι όλοι περιττοί, \pmod 8 είναι 3 \equiv 7 \pmod 8 άτοπο.

Άρα, δύο είναι άρτιοι και ένας περιττός, οπότε αν m περιττός, είναι k^2+l^2 \equiv 6 \pmod 8 που είναι εύκολα άτοπο.


Κερδίζουμε ό,τι τολμούμε!
Άβαταρ μέλους
Ορέστης Λιγνός
Δημοσιεύσεις: 1835
Εγγραφή: Κυρ Μάιος 08, 2016 7:19 pm
Τοποθεσία: Χαλάνδρι Αττικής
Επικοινωνία:

Re: JBMO Τέστ Εξάσκησης #2

#5

Μη αναγνωσμένη δημοσίευση από Ορέστης Λιγνός » Σάβ Ιουν 15, 2019 11:08 pm

achilleas έγραψε:
Σάβ Ιουν 15, 2019 2:24 pm

ΘΕΜΑ 3. Να δειχθεί ότι αν οι a,b,c είναι θετικοί αριθμοί τέτοιοι ώστε a^2+b^2+c^2=3, τότε \displaystyle  
	\dfrac{1}{a}+\dfrac{3}{b}+\dfrac{5}{c}\geq 4a^2+3b^2+2c^2.
Πότε ισχύει η ισότητα?
Από το Tangent Line Trick είναι :

\dfrac{1}{a}-4a^2 \geqslant \dfrac{3-9a^2}{2}

\dfrac{3}{b}-3b^2 \geqslant \dfrac{9-9b^2}{2}

\dfrac{5}{c}-2c^2 \geqslant \dfrac{15-9c^2}{2}

Προσθέτοντας κατά μέλη το ζητούμενο γίνεται άμεσο.


Κερδίζουμε ό,τι τολμούμε!
Άβαταρ μέλους
Ορέστης Λιγνός
Δημοσιεύσεις: 1835
Εγγραφή: Κυρ Μάιος 08, 2016 7:19 pm
Τοποθεσία: Χαλάνδρι Αττικής
Επικοινωνία:

Re: JBMO Τέστ Εξάσκησης #2

#6

Μη αναγνωσμένη δημοσίευση από Ορέστης Λιγνός » Σάβ Ιουν 15, 2019 11:14 pm

achilleas έγραψε:
Σάβ Ιουν 15, 2019 2:24 pm

ΘΕΜΑ 4. Σε n διαφανή κουτιά υπάρχουν κόκκινες και μπλε μπάλες. Θέλουμε να επιλέξουμε 50 κουτιά τέτοια ώστε να περιέχουν, μαζί, τουλάχιστον τις μισές κόκκινες και τουλάχιστον τις μισές μπλε μπάλες. Είναι μια τέτοια επιλογή δυνατή ανεξάρτητα από τον αριθμό των μπαλών και από την κατανομή τους στα κουτιά, αν

(α) n=100?
(β) n=99?

(α) Όχι. Τοποθετώντας από 1 κόκκινη μπάλα σε 25 κουτιά και 1 μπλε σε 75 κουτιά, χρειαζόμαστε τουλάχιστον 13+38 =51 κουτιά για μία κατάλληλη επιλογή, άτοπο.

(β) Ναι. Έστω r_i ο αριθμός των κόκκινων μπαλών σε κάθε κουτί, και WLOG r_i>r_j όταν i>j.

Επιλέγω το κουτί με αριθμό κόκκινων μπαλών r_1.

Από τα r_2,r_3 επιλέγω αυτό με τον μεγαλύτερο αριθμό μπλε μπαλών.

Όμοια για τα r_4,r_5 και συνεχίζω έτσι.

Στο τέλος, έχω επιλέξει ακριβώς 50 κουτιά, και είναι προφανές ότι αυτή η επιλογή περιέχει τουλάχιστον τις μισές μπλε και τις μισές κόκκινες μπάλες.


Κερδίζουμε ό,τι τολμούμε!
minageus
Δημοσιεύσεις: 18
Εγγραφή: Σάβ Μάιος 25, 2019 7:28 pm

Re: JBMO Τέστ Εξάσκησης #2

#7

Μη αναγνωσμένη δημοσίευση από minageus » Κυρ Ιουν 16, 2019 1:03 pm

Ορέστης Λιγνός έγραψε:
Σάβ Ιουν 15, 2019 11:06 pm
achilleas έγραψε:
Σάβ Ιουν 15, 2019 2:24 pm
Καλησπέρα σας!

Σε συνέχεια του προηγούμενου θέματος, ακολουθούν τα προβλήματα του 2ου τεστ.

Θα χαρούμε να δούμε κι άλλες διαφορετικές λύσεις στα παρακάτω θέματα.

**********************************************



ΘΕΜΑ 2. Να δειχθεί ότι η εξίσωση \displaystyle  
	x^2+y^2+z^2=x+y+z+1 
	δεν έχει ρητές λύσεις.
Γνωστό θέμα.

Ισοδύναμα, θέλω η (2x-1)^2+(2y-1)^2+(2z-1)^2=7 να μην έχει ρητές λύσεις, ή θέτοντας 2x-1=k/q, 2y-1=\ell/q, 2z-1=m/q με (k,\ell,m,q)=1 αρκεί να δείξω ότι η k^2+\ell^2+m^2=7q^2 δεν έχει ακέραιες λύσεις.

Αν q άρτιος, τότε πρέπει αναγκαστικά δύο εκ των k,\ell,m να είναι περιττοί και ένας άρτιος. Άρα, \pmod 8, LHS \equiv 2 ή 6 \pmod 8 και RHS \equiv 7 ή 28 \pmod 8, άτοπο.

Αν q περιττός, τότε αν οι k,\ell,m είναι όλοι περιττοί, \pmod 8 είναι 3 \equiv 7 \pmod 8 άτοπο.

Άρα, δύο είναι άρτιοι και ένας περιττός, οπότε αν m περιττός, είναι k^2+l^2 \equiv 6 \pmod 8 που είναι εύκολα άτοπο.
Συγκεκριμένα ,το δεύτερο θέμα έχει τεθεί στην Μεσογειάδα το 2003.


Δημήτρης Μηνάγιας
Απάντηση

Επιστροφή σε “Θέματα διαγωνισμών (ΕΜΕ, ΚΥΜΕ, BMO, JBMO, IMO, Kangaroo κλπ)”

Μέλη σε σύνδεση

Μέλη σε αυτήν τη Δ. Συζήτηση: Δεν υπάρχουν εγγεγραμμένα μέλη και 14 επισκέπτες